Last visit was: 26 Apr 2024, 17:07 It is currently 26 Apr 2024, 17:07

Close
GMAT Club Daily Prep
Thank you for using the timer - this advanced tool can estimate your performance and suggest more practice questions. We have subscribed you to Daily Prep Questions via email.

Customized
for You

we will pick new questions that match your level based on your Timer History

Track
Your Progress

every week, we’ll send you an estimated GMAT score based on your performance

Practice
Pays

we will pick new questions that match your level based on your Timer History
Not interested in getting valuable practice questions and articles delivered to your email? No problem, unsubscribe here.
Close
Request Expert Reply
Confirm Cancel
SORT BY:
Kudos
avatar
Intern
Intern
Joined: 04 Mar 2012
Posts: 35
Own Kudos [?]: 1323 [27]
Given Kudos: 10
Send PM
Most Helpful Reply
Board of Directors
Joined: 11 Jun 2011
Status:QA & VA Forum Moderator
Posts: 6072
Own Kudos [?]: 4690 [9]
Given Kudos: 463
Location: India
GPA: 3.5
WE:Business Development (Commercial Banking)
Send PM
CEO
CEO
Joined: 24 Jul 2011
Status: World Rank #4 MBA Admissions Consultant
Posts: 3187
Own Kudos [?]: 1585 [6]
Given Kudos: 33
GMAT 1: 780 Q51 V48
GRE 1: Q170 V170
Send PM
General Discussion
Current Student
Joined: 13 Apr 2015
Posts: 1436
Own Kudos [?]: 4548 [3]
Given Kudos: 1228
Location: India
Send PM
Re: In a hostel, the number of students decreased by 8% and the [#permalink]
3
Kudos
stonecold wrote:
Vyshak
Any Shortcuts here ?
took me 8 minutes to solve
i solved twice
actually thrice :)

regards
Stone Cold


Hi,

Assume there are 100 students, price/student = 10, Consumption = 1unit --> Cost = 100*10*1 = 1000
Given scenario: 92 students, price/student = 12, Consumption = x, But cost = 1000

92*12*x = 1000
x = 1000/1104 = 125/138 = 0.9 (approx)
So consumption has reduced by (1 - 0.9)*100 = 10% (approx)

Answer: E
Manager
Manager
Joined: 23 May 2013
Posts: 170
Own Kudos [?]: 402 [2]
Given Kudos: 42
Location: United States
Concentration: Technology, Healthcare
GMAT 1: 760 Q49 V45
GPA: 3.5
Send PM
Re: In a hostel, the number of students decreased by 8% and the [#permalink]
2
Kudos
I cannot see this being a GMAT problem - how is one supposed to calculate 1- [1/(.92*1.2)] without a calculator? Even with using fractions I get 13/138 which is also not easily solved.
User avatar
Retired Moderator
Joined: 16 Jun 2012
Posts: 871
Own Kudos [?]: 8554 [2]
Given Kudos: 123
Location: United States
Send PM
Re: In a hostel, the number of students decreased by 8% and the [#permalink]
1
Kudos
1
Bookmarks
PareshGmat wrote:
speedilly wrote:
I cannot see this being a GMAT problem - how is one supposed to calculate 1- [1/(.92*1.2)] without a calculator? Even with using fractions I get 13/138 which is also not easily solved.


Agree.. this calculation goes tough. Any simpler way to avoid that 92 & 12 factor?


Hello

My approach is

Name:
S = number of students
P = Price
Q = Quantity of food consumption.
C = total cost

==> C = S*P*Q

One way to solve this kind of question is plug in numbers.
Assume: before we have 100 students, P = $10 ==> C = 100*10*Q = 1000*Q
After: we have 92 students, P = $12 ==> C' = 92*12*Q' = 1104*Q'

==> 1104*Q' = 1000*Q
==> Q' = 1000/1104*Q = (1 - 104/1104)*Q
==> Q' = Q - 104/1104*Q

The red part is the decrease of food consumption. We can guess 104/1104 ~ 9.4% (The ONLY option < 10%) --> E is answer.

Hope it helps.
avatar
SVP
SVP
Joined: 27 Dec 2012
Status:The Best Or Nothing
Posts: 1562
Own Kudos [?]: 7208 [1]
Given Kudos: 193
Location: India
Concentration: General Management, Technology
WE:Information Technology (Computer Software)
Send PM
Re: In a hostel, the number of students decreased by 8% and the [#permalink]
1
Kudos
speedilly wrote:
I cannot see this being a GMAT problem - how is one supposed to calculate 1- [1/(.92*1.2)] without a calculator? Even with using fractions I get 13/138 which is also not easily solved.


Agree.. this calculation goes tough. Any simpler way to avoid that 92 & 12 factor?
Alum
Joined: 12 Aug 2015
Posts: 2282
Own Kudos [?]: 3132 [0]
Given Kudos: 893
GRE 1: Q169 V154
Send PM
Re: In a hostel, the number of students decreased by 8% and the [#permalink]
Vyshak
Any Shortcuts here ?
took me 8 minutes to solve
i solved twice
actually thrice :)

regards
Stone Cold
Senior Manager
Senior Manager
Joined: 02 Apr 2014
Posts: 371
Own Kudos [?]: 474 [0]
Given Kudos: 1227
Location: India
Schools: XLRI"20
GMAT 1: 700 Q50 V34
GPA: 3.5
Send PM
Re: In a hostel, the number of students decreased by 8% and the [#permalink]
Let there be 100 students , price of each food student be 100$
Now 8% number of students reduced = 92, price of food increased by 20% = 120$
x(x is decimal less than 1) be reduced consumption of food, so price of reduced food = 120x

given 100 * 100 = 92 * 120 * x
x = 10000/(92*120) = 1000/1104 = 10/11 (approx)
so how much each student has to reduce = 1 - 10/11 = 1/11 = 9% approx => closest is (E)
SVP
SVP
Joined: 24 Nov 2016
Posts: 1720
Own Kudos [?]: 1344 [0]
Given Kudos: 607
Location: United States
Send PM
Re: In a hostel, the number of students decreased by 8% and the [#permalink]
GyanOne wrote:
Cost of food (C) = Food consumed per student (F) * Number of students (N) * Price of food (P)

Originally,
C = FNP

When number of students decrease by 8%, and the price of food increases by 20%,
C = F(new) * (0.92N) * (1.2P)
=> F(new) = F/(0.92*1.2)
=> F(new) = 0.906F

Therefore the new cost of food must be 90.6% of the old cost, or the cost of food must decrease by 9.4% (option E)


just to clear things up, what you did in the second part was:
C = F(new) * (0.92N) * (1.2P)
FNP = F(new) * (0.92N) * (1.2P)
F(new) = FNP / (0.92N) * (1.2P)
F(new) = F / (0.92) * (1.2)
F(new) = 0.906F

correct?
Intern
Intern
Joined: 13 Aug 2019
Posts: 27
Own Kudos [?]: 12 [0]
Given Kudos: 12
Location: India
Concentration: Strategy, Marketing
WE:Analyst (Retail)
Send PM
Re: In a hostel, the number of students decreased by 8% and the [#permalink]
Rule: If the overall/final value is not changing and if one variable is increased by \(\frac{1}{x}\)% then the other variable should decrease by \(\frac{1}{x+1}\)%
for eg: if AB = 100, now if B is increased by \(33.33\)% then by how much percentage should A decrease such that the final value remains the same.
Now, B is increased by \(33.33\)% --> \(\frac{1}{3}\)% --> \(x=3\)
Then if final value is same, i.e; 100, then A should decrease by \(\frac{1}{x+1}\)% = \(\frac{1}{(3+1)}\)% = \(\frac{1}{4}\)% = \(25\)%

To be more clear - let \(A = 20\) and\( B = 5\) --> \(AB = 20*5\) = \(100\)
Now if \(B = 5\) is increased by \(33.33\)% = \(5*1.3333\) = \(6.6665\), then \(A = 20\) should decrease by \(25\)% = \(20*0.75\) = \(15\),
such that \(AB = 6.6665*15\) = \(99.997\) =\(~100\) (same as before)

We can apply the same logic here
Let say, Total consumption = \(T\)
consumption of food / student = \(c\)
total number of students = \(n\)
price per food = \(p\)

Hence,
\(T = c * n * p\)

Let us take \(n * p = F\)

So, \(T = c * F\)

Now, \(n\) is decreased by \(8\)% and \(p\) is increased by \(20\)%, we can calculate overall % change in \(F\) by formula \(A + B + \frac{AB}{100}\)
overall % change in \(F\) = \(20 - 8 - \frac{20(8)}{100}\) = \(10.4\)%

Total consumption(after change), \(T = c' * 10.4F\), where \(c'\) = change in the consumption

Now, let's apply the above logic we learned,

The total consumption \(T\) is same and one variable \(F\)(here) is increased by \(10.4\)% ~ \(10\)%(approximate) = \(\frac{1}{10}\)% = \(x = 10\)

Hence the other variable \(c'\) (here) should decrease by \(\frac{1}{x+1}\)% = \(\frac{1}{(10+1)}\)% = \(\frac{1}{11}\)% = \(9.09\)% ~ \(9\)%

The only answer option closest to \(9\)% = \(9.4\)% Option E

Kudos if find solution helpful :)
User avatar
Non-Human User
Joined: 09 Sep 2013
Posts: 32688
Own Kudos [?]: 822 [0]
Given Kudos: 0
Send PM
Re: In a hostel, the number of students decreased by 8% and the [#permalink]
Hello from the GMAT Club BumpBot!

Thanks to another GMAT Club member, I have just discovered this valuable topic, yet it had no discussion for over a year. I am now bumping it up - doing my job. I think you may find it valuable (esp those replies with Kudos).

Want to see all other topics I dig out? Follow me (click follow button on profile). You will receive a summary of all topics I bump in your profile area as well as via email.
GMAT Club Bot
Re: In a hostel, the number of students decreased by 8% and the [#permalink]
Moderators:
Math Expert
92948 posts
Senior Moderator - Masters Forum
3137 posts

Powered by phpBB © phpBB Group | Emoji artwork provided by EmojiOne